LSAT and Law School Admissions Forum

Get expert LSAT preparation and law school admissions advice from PowerScore Test Preparation.

 Luke Haqq
PowerScore Staff
  • PowerScore Staff
  • Posts: 747
  • Joined: Apr 26, 2012
|
#41892
Hi puppytiff!

In choosing between (B) and (A), one thing that is important to remember is that this strengthen question involves cause-and-effect reasoning.

In the end, what (B) seems to do is it just reiterates a premise--but it doesn't get to the cause and effect reasoning. Namely, "Deer are invading residential areas, damaging property and causing motor vehicle accidents that result in serious injury to motorists." is unpacked as "Motor vehicle accidents involving deer often result in damage to the vehicle, injury to the motorist, or both." In other words, (B) is repeating the part about the "injury to the motorist," and it's presumably the case that such an accident would also be "damaging property." So, theoretically, something like (B) might be a contender inasmuch as it provides new information that reinforces a premise, but that's unlikely because only supporting a premise won't necessarily get to the argument.

(A) also focuses on a couple premises, namely, "(1) Eight years ago hunting was banned in Greenfield County on the grounds that hunting endangers public safety. (2) Now the deer population in the county is six times what it was before the ban." (A) uses other counties to support that data--but it's not only supporting those premises on their own, it's supporting the implicit cause-and-effect relationship in those sentences, the relationship the conclusion is making an argument about.

(A) states, "In surrounding counties, where hunting is permitted, the size of the deer population has not increased in the last eight years." It's initially possible that the deer population rose to six times what it was before the ban, but that the rise in population had nothing to do with the ban. Answer (A) denies that possibility--it reiterates the cause-and-effect relationship by claiming, based on data from surrounding counties, it was indeed the ban that caused the increase in population. Given that (A) reinforces that, it thus reinforces the conclusion that Greenfield's ban was both unnecessary and created a danger to public safety that otherwise would not exist.

Hope that helps!
 lunsandy
  • Posts: 61
  • Joined: Oct 14, 2017
|
#42562
Hi Powerscore,

Upon reviewing, I noticed for both this question L5-68 Qs. 31 and L5.65 Qs. 20 (in-person full course) I fell for the answer that is already stated in the stimulus (B for Qs. 31 and A for Qs. 20, respectively). Can it be said that if an answer choice repeats what is already mentioned in the stim then that answer does NOT strengthen the conclusion of the argument because it already has been stated, thus does nothing to the argument.

So, for B, is wrong, as mentioned previously, because "motor vehicle accidents involving deer often result in damage to vehicle" is mentioned in the stim. as "deer are invading residential areas...and causing motor vehicle accidents" thus by repeating what is already said does nothing to our conclusion.

Does this follow for all strengthening question? Is this a trap answer that I should take note for future strengthening questions?

Thanks a lot!
 James Finch
PowerScore Staff
  • PowerScore Staff
  • Posts: 943
  • Joined: Sep 06, 2017
|
#42592
Hi Lunsandy,

Yes, that's exactly what Luke was getting at. If we already have a stated premise in the conclusion, then a restatement of that premise can't strengthen our conclusion; it's already supporting the conclusion as-is! Instead, this type of question (Family #2) is looking for another outside premise to bring into the argument, and in this case, strengthen it. The key to being able to answer these efficiently and accurately is to recognize the type of reasoning in the stimulus and be on the lookout for the answer choice that would strengthen that reasoning. With Strengthen and Weaken questions, there is always a logical gap that needs to be filled in (or made wider in the Weaken case) to support the conclusion.

Here we have a causal argument, that the hunting ban has led to more deer, which in turn has led to a safety risk. The stated premises make a solid link between more deer and safety issues, but we don't have a control group to show that the hunting ban is what actually caused the explosion in the deer population. Answer choice (A) addresses this by bringing in a new premise that surrounding counties (ie similar to the one in the stimulus) that have continued to allow deer hunting have not seen their deer populations change over the period that the hunting ban has been in effect. This is strong evidence that the hunting ban is the cause of the growth in the deer population, and thus strengthens our argument, making it the correct answer choice.

Hope this helps!
 lunsandy
  • Posts: 61
  • Joined: Oct 14, 2017
|
#42593
Hi James,

Thanks for your reply, your explanation was very helpful!

I disregarded A at first because I thought it was out of scope, but as you said for strengthening and weakening questions we often do need new information to fill in the gap of our argument (unlike the first family MSS/MBT). Thank you :)
User avatar
 teddykim100
  • Posts: 46
  • Joined: Jan 10, 2022
|
#96872
I was thrown off primarily by how the direct premise before the conclusion is “Since there were never any hunting related injuries in the county. . .”

My thought process directed me to think that if we were to change the type of question to a “weaken”, the flaw I would find in the argument is that the ban could have been unnecessary because of OTHER reasons than the hunting related injuries. AKA you can’t just attribute the “unnecessary-ness” of the ban to hunting injuries. AKA the ban could be unnecessary for other reasons, why just assume it’s ONLY the hunting injuries?

Maybe it’s unnecessary because it increases taxes, puts firearms dealers out of business, makes people move out of town, etc.

As such, it didn’t occur to me to look for a gap in logic that would address the population.

Please let me know if I said anything unclear, and thank you for the help!
 Rachael Wilkenfeld
PowerScore Staff
  • PowerScore Staff
  • Posts: 1358
  • Joined: Dec 15, 2011
|
#96889
Hi teddy,

The conclusion here is more than the idea that the hunting ban was unnecessary. The conclusion also includes the idea that the ban created (or caused) a danger to public safety that did not exist before. So the ban is both unnecessary (it doesn't serve a need) and creates a danger to safety. The examples you are listing with taxation loss and putting companies out of business are not related to public safety, or reasons the ban would be unnecessary. They are reasons that you might not support a ban, but they are not related to the specific conclusion of this stimulus.

We are trying to strengthen the idea that the ban is a danger to public safety. Answer choice (A) does this by showing that in areas where hunting is not banned, the deer population has not increased, and thus the negative effects of the increase would not be expected without the ban. We'd call that no cause, no effect, which is one way to strengthen a causal argument.

Hope that helps!

Get the most out of your LSAT Prep Plus subscription.

Analyze and track your performance with our Testing and Analytics Package.